LSAT and Law School Admissions Forum

Get expert LSAT preparation and law school admissions advice from PowerScore Test Preparation.

User avatar
 Dave Killoran
PowerScore Staff
  • PowerScore Staff
  • Posts: 5853
  • Joined: Mar 25, 2011
|
#41281
Complete Question Explanation
(The complete setup for this game can be found here: lsat/viewtopic.php?t=11660)

The correct answer choice is (C)

This question is almost the same as #2, except now the focus is specifically on day 5. According to our Not Laws, J cannot be inspected on day 5, and so it must be that answer choice (C) is correct. This is an excellent example of what “attacking the question” means. By using the Not Laws, you should already know what the answer is before looking at the answer choices. Instead of slowly looking at answer choice (A), then (B), and so on, why not immediately look for “J” as one of the choices? This saves time, and more importantly, builds your confidence as you find the answer that you are looking for.

An interesting aside: the worst answer choice for a student to select in this question is answer choice (E). Why? Because from the hypothetical we produced in question #1 we know that R can be scheduled for day 5. Even if you cannot figure out which answer was definitely correct in this problem, you could use that hypothetical to kill answer choice (E) and improve your odds of answering correctly. And, although this information is not of very great value on this question, in future games this technique will prove to be very helpful indeed.
 lakasil
  • Posts: 10
  • Joined: Aug 08, 2016
|
#28536
On page 161-163 Why is (C) proven to be the correct answer? I understand the books explanation, but there's no not laws under slot 2 stating that " J" can't go there?
 Nikki Siclunov
PowerScore Staff
  • PowerScore Staff
  • Posts: 1362
  • Joined: Aug 02, 2011
|
#28553
Hi lakasil,

A J Not-Law on day 2 is not indicated, because J can be on day 2. In fact, the correct answer choice (C) places J on day 2. Please clarify why you believe that J cannot be on day 2.

In case you're wondering why the diagram on page 163 is impossible, it's not because J happens to be on day 2. As explained immediately above and below the diagram, in this setup G is inspected on day 3, but Q is not inspected on day 5—a clear violation of the last rule. J's position in this solution has no bearing on its acceptability.

Thanks,

Get the most out of your LSAT Prep Plus subscription.

Analyze and track your performance with our Testing and Analytics Package.